LSAT and Law School Admissions Forum

Get expert LSAT preparation and law school admissions advice from PowerScore Test Preparation.

User avatar
 Dave Killoran
PowerScore Staff
  • PowerScore Staff
  • Posts: 5853
  • Joined: Mar 25, 2011
|
#94166
Complete Question Explanation
(The complete setup for this game can be found here: viewtopic.php?f=393&t=1212)

The correct answer choice is (C)

The question stem establishes the following information:

G4-Q23-d1.png

If N, O, and T participate in the first year, then S and P must participate in the second year. Because P participates in the second year and the fourth year, from the second rule we can infer that P does not participate in the third year:

G4-Q23-d2.png

Thus, answer choice (C) is correct.
You do not have the required permissions to view the files attached to this post.

Get the most out of your LSAT Prep Plus subscription.

Analyze and track your performance with our Testing and Analytics Package.